n is an integer greater than or equal to 0. The sequence t_n

This topic has expert replies
Legendary Member
Posts: 944
Joined: Wed May 30, 2012 8:21 am
Thanked: 8 times
Followed by:5 members
n is an integer such that n>= 0. For n > 0, the sequence t_n is defined as t_n = t_{n-1} + n. Given that t_0 = 3, is t_n even?

(1) n + 1 is divisible by 3
(2) n - 1 is divisible by 4


OA : B

@ Experts - How we can solve this in least possible time and is there any other way to tackle this type of problems rather than plugging in values & subsequently observing the pattern ?

User avatar
GMAT Instructor
Posts: 15539
Joined: Tue May 25, 2010 12:04 pm
Location: New York, NY
Thanked: 13060 times
Followed by:1906 members
GMAT Score:790

by GMATGuruNY » Tue Mar 31, 2015 4:34 am
RBBmba@2014 wrote:n is an integer such that n>= 0. For n > 0, the sequence t_n is defined as t_n = t_{n-1} + n. Given that t_0 = 3, is t_n even?

(1) n + 1 is divisible by 3
(2) n - 1 is divisible by 4
WRITE OUT a few terms and LOOK FOR A PATTERN.

tâ‚€ = 3.
t� = t₀ + 1 = 3 + 1 = 4.
t₂ = t� + 2 = 4 + 2 = 6.
t₃ = t₂ + 3 = 6 + 3 = 9.
t₄ = t₃ + 4 = 9 + 4 = 13.
tâ‚… = tâ‚„ + 5 = 13 + 5 = 18.
t₆ = t₅ + 6 = 18 + 6 = 24.
t₇ = t₆ + 7 = 24 + 7 = 31.
t₈ = t₇ + 8 = 31 + 8 = 39.

The resulting values after tâ‚€ -- 4, 6, 9, 13, 18, 24, 31, 39 -- imply the following pattern:
EVEN, EVEN, ODD, ODD, EVEN, EVEN, ODD, ODD...

Statement 1: n + 1 is divisible by 3
Statement 1 is satisfied by the options in red:
tâ‚€ = 3.
t� = t₀ + 1 = 3 + 1 = 4.
t₂ = t� + 2 = 4 + 2 = 6.
t₃ = t₂ + 3 = 6 + 3 = 9.
t₄ = t₃ + 4 = 9 + 4 = 13.
tâ‚… = tâ‚„ + 5 = 13 + 5 = 18.
t₆ = t₅ + 6 = 18 + 6 = 24.
t₇ = t₆ + 7 = 24 + 7 = 31.
t₈ = t₇ + 8 = 31 + 8 = 39.
Since t_n can be EVEN or ODD, INSUFFICIENT.

Statement 2: n - 1 is divisible by 4
Statement 2 is satisfied by the options in blue:
tâ‚€ = 3.
t� = t₀ + 1 = 3 + 1 = 4.
t₂ = t� + 2 = 4 + 2 = 6.
t₃ = t₂ + 3 = 6 + 3 = 9.
t₄ = t₃ + 4 = 9 + 4 = 13.
tâ‚… = tâ‚„ + 5 = 13 + 5 = 18.
t₆ = t₅ + 6 = 18 + 6 = 24.
t₇ = t₆ + 7 = 24 + 7 = 31.
t₈ = t₇ + 8 = 31 + 8 = 39.
t₉ = EVEN.
t�₀ = EVEN.
t�� = ODD.
t�₂ = EVEN.
t�₃ = EVEN.
In every case, t_n is EVEN.
SUFFICIENT.

The correct answer is B.
Private tutor exclusively for the GMAT and GRE, with over 20 years of experience.
Followed here and elsewhere by over 1900 test-takers.
I have worked with students based in the US, Australia, Taiwan, China, Tajikistan, Kuwait, Saudi Arabia -- a long list of countries.
My students have been admitted to HBS, CBS, Tuck, Yale, Stern, Fuqua -- a long list of top programs.

As a tutor, I don't simply teach you how I would approach problems.
I unlock the best way for YOU to solve problems.

For more information, please email me (Mitch Hunt) at [email protected].
Student Review #1
Student Review #2
Student Review #3

Legendary Member
Posts: 944
Joined: Wed May 30, 2012 8:21 am
Thanked: 8 times
Followed by:5 members

by RBBmba@2014 » Tue Mar 31, 2015 6:45 am
Thanks Mitch for your reply.

I did it in the same way but I landed up using 3+ minutes, so I thought whether there is any smarter way to solve this ?

User avatar
GMAT Instructor
Posts: 15539
Joined: Tue May 25, 2010 12:04 pm
Location: New York, NY
Thanked: 13060 times
Followed by:1906 members
GMAT Score:790

by GMATGuruNY » Wed Apr 01, 2015 3:08 am
RBBmba@2014 wrote:Thanks Mitch for your reply.

I did it in the same way but I landed up using 3+ minutes, so I thought whether there is any smarter way to solve this ?
tâ‚€ = 3.
t� = t₀ + 1 = 3 + 1 = 4.
t₂ = t� + 2 = 4 + 2 = 6.
t₃ = t₂ + 3 = 6 + 3 = 9.
t₄ = t₃ + 4 = 9 + 4 = 13.
tâ‚… = tâ‚„ + 5 = 13 + 5 = 18.
t₆ = t₅ + 6 = 18 + 6 = 24.
t₇ = t₆ + 7 = 24 + 7 = 31.
t₈ = t₇ + 8 = 31 + 8 = 39.

The sequence is composed of a repeating 4-term cycle:
EVEN, EVEN, ODD, ODD.
As indicated by the options in red, the cycle ENDS when n is a MULTIPLE OF 4.
The end of a cycle is always ODD.
The beginning of a cycle is always EVEN.

Statement 1: n + 1 is divisible by 3
Options for n+1 = 3, 6, 9, 12...
Options for n = 2, 5, 8, 11...

Case 1: n=5
Since tâ‚„ is the END of a cycle, tâ‚… is the BEGINNING of a new cycle, with the result that tâ‚… is EVEN.

Case 2: n=8
Since t₈ is the END of a cycle, t₈ is ODD.
INSUFFICIENT.

Statement 2: n - 1 is divisible by 4
Since t_(n-1) is the END of a cycle, t_n is the BEGINNING of a new cycle, with the result that t_n is EVEN.
SUFFICIENT.

The correct answer is B.
Private tutor exclusively for the GMAT and GRE, with over 20 years of experience.
Followed here and elsewhere by over 1900 test-takers.
I have worked with students based in the US, Australia, Taiwan, China, Tajikistan, Kuwait, Saudi Arabia -- a long list of countries.
My students have been admitted to HBS, CBS, Tuck, Yale, Stern, Fuqua -- a long list of top programs.

As a tutor, I don't simply teach you how I would approach problems.
I unlock the best way for YOU to solve problems.

For more information, please email me (Mitch Hunt) at [email protected].
Student Review #1
Student Review #2
Student Review #3